A woman stands on a bathroom scale in a motionless elevator. When the elevator begins to move, the scale briefly reads only 0.64 of her regular weight. Calculate the magnitude of the acceleration of the elevator.

Answers

Answer 1

Answer:

The downwards acceleration is 3.53 m/s2.

Explanation:

Let the true weight is m g.

The reading of the balance, R = 0.64 mg

Let the acceleration is a.

As the apparent weight is less than the true weight so the elevator goes down wards with some acceleration.

Use Newton's second law

m g - R = m a

m g - 0.64 m g = m a

0.36 g = a

a = 3.53 m/s2


Related Questions

Three forces are pulling on the same object such that the system is in equilibrium. Their magnitudes are F1 = 2.83 N.F= 3.35 N. and F3 = 3.64 N, and they make angles of 0, = 45.0°, 02 = -63.43 and 03 =164.05° with respect to the x-axis, respectively.

Required:
a. What is the x-component of the force vector F1?
b. What is the y-component of the force vector F1?

Answers

Answer:

(a) 2.001N

(b) 2.001N

Explanation:

A sketch of the scenario has been attached to this response.

Since only the force vector F₁ is required, the only force shown in the sketch is F₁.

As shown in the sketch;

The x-component of the force vector F₁ = [tex]F_{x}[/tex]

The y-component of the force vector F₁ = [tex]F_{y}[/tex]

The magnitude of F₁ as given in the question = 2.83N

The angle that the force makes with respect to the x-axis = 45.0°

Using the trigonometric ratio, we see that;

(a) cos 45.0° = [tex]\frac{F_x}{F_1}[/tex]

=>  [tex]F_{x}[/tex] =  F₁ cos 45.0°

=> [tex]F_{x}[/tex] =  2.83 cos 45.0°

=> [tex]F_{x}[/tex] =  2.83 x 0.7071

=> [tex]F_{x}[/tex] =  2.001N

(b) Also;

sin 45.0° = [tex]\frac{F_y}{F_1}[/tex]

=>   [tex]F_{y}[/tex] =  F₁ sin 45.0°

=>  [tex]F_{y}[/tex] =  2.83 sin 45.0°

=>  [tex]F_{y}[/tex] =  2.83 x 0.7071

=>  [tex]F_{y}[/tex] =  2.001N

Therefore, the x-component and y-component of the force vector F₁ is 2.001N

The x and y component of vector F1 is mathematically given as

F_x =  2.001N

F_y=  2.001N

What is the x and y component of vector F1?

Question Parameters:

Generally, the equation for the x-component  is mathematically given as

x=Fsin\theta

Therefore

F_x =  F₁ cos 45.0°

F_x =  2.83 x 0.7071

F_x =  2.001N

For y component

x=Fcos\theta

F_y =  F₁ sin 45.0

F_y =  2.83 x 0.7071

F_y=  2.001N

Read more about Cartesian

https://brainly.com/question/9410676

A balloon is filled with 80 liters of gas on a day where the temperature was 34 degrees at sea level which is 101.3 kPa and released. As the balloon rises to a certain altitude, the temperature drops to 0 degrees celsius and the balloon doubles in volume. What is the atmospheric pressure at that altitude?

Answers

Answer:

0.444atm

Explanation:

Using the combined gas law equation;

P1V1/T1 = P2V2/T2

Where;

P1 = initial pressure (

P2 = final pressure (

V1 = initial volume (L)

V2 = final volume (L)

T1 = initial temperature (K)

T2 = final temperature (K)

According to this question,

P1 = 101.3 kPa = 101.3 × 0.00987 = 0.999atm

P2 = ?

V1 = 80L

V2 = 160L (double of V1)

T1 = 34°C = 34 + 273 = 307K

T2 = 0°C = 0 + 273 = 273K

Using P1V1/T1 = P2V2/T2

0.999 × 80/307 = P2 × 160/273

79.92/307 = 160P2/273

Cross multiply

307 × 160P2 = 79.92 × 273

49120P2 = 21818.16

P2 = 21818.16 ÷ 49120

P2 = 0.444

P2 = 0.444atm

Question 2:
Inclined Plane
A block (M) weighs 25-N, rests on an inclined plane when it is joined by a sting to a support
(S) as shown in the figure' below. Use g=10 N/Kg.
(S)
B
M
List and classify the forces acting on (M).
Représent, without scaling, the forces acting on (M).
Find the mass of (M).
74. If the string were cut, (M) does not slide. Explain this phenomenon.
15. Determine the mass and weight of (M) on moon.
06​

Answers

Answer:

we need the block

Explanation:

1×2 =4 lest 74 =345

A 92-kg man climbs into a car with worn out shock absorbers, and this causes the car to drop down 4.5 cm. As he drives along he hits a bump, which starts the car oscillating at an angular frequency of 4.52 rad/s. What is the mass of the car ?A) 890 kg
B) 1900 kg
C) 920 kg
D) 990 kg
E) 760 kg

Answers

Answer:

the mass of the car is 890 kg

Explanation:

Given;

mass of the man, m = 92 kg

displacement of the car's spring, x = 4.5 cm = 0.045 m

acceleration due to gravity, g = 9.8 m/s²

The spring constant of the car,

f = kx

where;

f is the weight of the man on the car = mg

mg = kx

k = mg/x

k = (92 x 9.8) / 0.045

k = 20,035.56 N/m

The angular speed of car, ω, when the is inside is given as 4.52 rad/s

The total mass of the car and the man is calculated as;

[tex]\omega = \sqrt{\frac{k}{m} } \\\\\omega^2 = \frac{k}{m} \\\\m = \frac{k}{\omega^2} = \frac{20,035.56}{(4.52)^2} = 980.7 \ kg[/tex]

The mass of the car alone = 980.7 kg - 92 kg

                                            = 888.7 kg

                                             ≅ 890 kg

Therefore, the mass of the car is 890 kg

which unit would be most suitable for its scale?
A mm
B
с
crn?
D
cm
[0625_504_9p_1].
8
A piece of cotton is measured between two points on a ruler.
1
coton
BAS
2
4
5
6
7
8
9
10
11
12
13
14
15 16
when the lenge of coton is wound closely around a pen, goes round six times.
pen
six turns of coton
दे-
What is the distance onde round the pen?
4 2.2 m
B 26 cm
с
13.2 cm
D 15.6 cm

Answers

Answer:

Mm, thats the answer trust me men

If R1 and R2 are in parallel and R3 is in series with them then equivalent resistance will be

Answers

Answer:

Refer to the attachment!~

The slope at point A of the graph given below is:


WILL MARK BRAINLIEST TO CORRECT ANSWER

Answers

RQ/PQ I think

rise/run

A car has a mass of 900 kg is accelerated from rest at a rate of 1.2 m/s calculate the time taken to reach 30/s​

Answers

Answer:

12+2=24+30+2=66

Explanation:

What is the current in the 30 resistor?
A. 0.0833 A
B. 12 A
C. 2 A
D. 10 A

Answers

Answer:

Explanation:

Step 1) Combine all resistors into an equivalent overall resistor. These are all in series so you just add them up. Req = 10Ω + 20Ω + 30Ω = 60Ω:

Step 2) Using Ohm's Law, I = V/R = 120/60 = 2 A

Now you know how much current is flowing, and that current flows through each resistor the same. So the current in the 30 Ω resistor is 2.00 amps.

A bucket of mass m in a well is held up by a rope. the rope is wound around a drum of radius r there is also a handle of length R attached to the drum. the tension in the rope is equal to T. If the buket is allowed to fall into the well, which point will have the greatest angular acceleration, a point on the rim of the drum (at radius r) or a point on the end of the handle (at radius R)?

a. The point on the rim of the drum.
b. The point at the end of the handle.
c. They will both have the same angular acceleration.

Answers

Answer:

the correct answer is C

Explanation:

This is a system with circular motion, there is a relationship between the linear and angular variables

         a = α r

with the cube going down the well, the tension of the leather is maintained therefore the acceleration of the cube is

          W = m a

          -mg = ma

          a = -g

this acceleration a is the same as that at the edge of the drum.

         α = a / r

where we can see that the angular acceleration is constant

consequently the correct answer is C

How is fitness walking beneficial?

It can relieve stress and improve mood.

It can decrease energy levels.

It can decrease perspiration.

It can relieve allergy symptoms.

Answers

Answer:

It can relieve stress and improve mood.

it can increase chances of a better lifestyle and better mental health

An exoplanet has three times the mass and one-fourth the radius of the Earth. Find the acceleration due to gravity on its surface, in terms of g, the acceleration of gravity at Earth's surface. A planet's gravitational acceleration is given by gp = G Mp/r^2p
a. 12.0 g.
b. 48.0 g.
c. 6.00 g.
d. 96.0 g.
e. 24.0 g.

Answers

Answer:

b. 48.0 g.

Explanation:

Given;

mass of the exoplanet, [tex]M_p = 3M_e[/tex]

radius of the exoplanet, [tex]r_p = \frac{1}{4} r_e[/tex]

The acceleration due to gravity of the planet is calculated as;

[tex]g_p = \frac{GM_p}{r_p^2} \\\\for \ Earth's \ surface\\\\g = \frac{GM_e}{r_e^2} \\\\G = \frac{gr_e^2}{M_e} = \frac{g_pr_p^2}{M_p} \\\\\frac{gr_e^2}{M_e} = \frac{g_p(\frac{r_e}{4}) ^2}{3M_e} \\\\\frac{gr_e^2}{M_e} = \frac{g_pr_e ^2}{16\times 3M_e} \\\\g = \frac{g_p}{48} \\\\g_p = 48 \ g[/tex]

Therefore, the correct option is b. 48.0 g

Mary applies a force of 73 N to push a box with an acceleration of 0.48 m/s^2. When she increases the pushing force to 84 N, the box's acceleration changes to 0.64 m/s^2. There is a constant friction force present between the floor and the box.

Required:
a. What is the mass of the box?
b. What is the coefficient of kinetic friction between the floor and the box?

Answers

Answer: [tex]68.75\ kg, 0.06[/tex]

Explanation:

Mary applies a force of 73 N to create an acceleration of [tex]0.48\ m/s^2[/tex]

When She increases force to 84 N, it creates an acceleration of [tex]0.64\ m/s^2[/tex]

Friction opposes the motion of box

[tex]\Rightarrow 73-f=m\times 0.48\quad \ldots(i)\\\Rightarrow 84-f=m\times 0.64\quad \ldots(ii)[/tex]

Subtract (i) from (ii)

[tex]\Rightarrow 11=m(0.64-0.48)\\\Rightarrow m=68.75\ kg[/tex]

Therefore friction is

[tex]\Rightarrow f=73-68.75\times 0.48\\\Rightarrow f=73-33\\\Rightarrow f=40\ N[/tex]

Here, friction is kinetic friction which is given by

[tex]\Rightarrow f=\mu_kmg\\\Rightarrow 40=\mu_k 68.75\times 9.8\\\Rightarrow \mu_k=0.061[/tex]

If ATM is 102 kPa, what force does the atmosphere exert on the palm of your hand which has an area of 0.016 meters?

Answers

Answer:

Force = 1.632 Newton

Explanation:

Given the following data;

Pressure = 102 kPa

Area = 0.016 m²

To find what force the atmosphere exert on the palm of your hand;

Mathematically, pressure is given by the formula;

[tex] Pressure = \frac {Force}{area} [/tex]

Force = 102 * 0.016

Force = 1.632 Newton

The equation of damped oscillations is given in the form x=0.05e^-0.25sin½πt (m). Find the velocity of an oscillating point at the moments of time: 0, T, 2T, 3T and 4T.​

Answers

Explanation:

Finding the (maximum) respective prime powers would yield the answer. Also we need not ... Is perfectly divisible by 720^n? ... So we can say that for any positive value of n it not divisible.

A 5.0-kg solid cylinder of radius 0.25 mis free to rotate about an axle that runs along the cylinders length and passes through its center. A thread wrapped around the cylinder is weighed down by a mass of 2.0 kg so as to unwrap and make the cylinder rotate as this mass falls. Ignore any friction in the axle. If there is no slippage between the thread and the cylinder, and the cylinder starts from rest (a) Calculate the velocity of the block after it has fallen a distance of 2.0m. Give your answer in m.s (b) Calculate the total work done by the rope on the cylinder after the block has fallen a distance of 2.0 m. Give your answer in Joule. ​

Answers

Answer:

157n is the correct answer

A meter stick has a mass of 0.30 kg and balances at its center. When a small chain is suspended from one end, the balance point moves 28.0 cm toward the end with the chain. Determine the mass of the chain.

Answers

Answer:

M L1 = m L2       torques must be zero around the fulcrum

M = m L2 / L1 = .3 kg * 28 cm / 22 cm = .382 kg

What is the electric field 3.9 m from the center of the terminal of a Van de Graaff with a 6.60 mC charge, noting that the field is equivalent to that of a point charge at the center of the terminal

Answers

Answer:

the electric field is  3.91 x 10⁶ N/C

Explanation:

Given the data in the question;

Electric field at a point due to point charge is;

E = kq/r²

where k is the constant, r is the distance from centre of terminal to point where electric field is, q is the excess charge placed on the centre of terminal of Van de Graff,a generator

Now, given that r = 3.9 m, k = 9.0 x 10⁹ Nm²/C², q = 6.60 mC = 6.60 x 10⁻³ C

so we substitute into the formula

E = [(9.0 x 10⁹ Nm²/C²)( 6.60 x 10⁻³ C)] / ( 3.9 )²

E = 59400000 / 15.21

E = 3.91 x 10⁶ N/C

Therefore, the electric field is  3.91 x 10⁶ N/C

They create a heat engine where the hot reservoir is filled with water and steam at equilibrium, and the cold reservoir is filled with ice and water at equilibrium. What is the Carnot efficiency for their heat engine if the pressure is constant at 1.0 atmospheres?

Answers

Answer:

The efficiency of Carnot's heat engine is 26.8 %.

Explanation:

Temperature of hot reservoir, TH = 100 degree C = 373 K

temperature of cold reservoir, Tc = 0 degree C = 273 K

The efficiency of Carnot's heat engine is

[tex]\eta = 1-\frac{Tc}{T_H}\\\\\eta = 1 -\frac{273}{373}\\\\\eta = 0.268 =26.8 %[/tex]

The efficiency of Carnot's heat engine is 26.8 %.

A 2 kg stone is dropped from a height of 100 m. How far does it travel in the third second? take g = 9.8 m/s2​

Answers

Answer:

S = 1/2 gt² = 1/2 × 9.8 × 3² = 4.9×9 = 44.1 m

Explanation:

A 75.0 kg diver falls from rest into a swimming pool from a height of 5.10 m. It takes 1.34 s for the diver to stop after entering the water. Find the magnitude of the average force exerted on the diver during that time.

Answers

Answer:

559.5 N

Explanation:

Applying,

v² = u²+2gs............. Equation 1

Where v = final velocity,

From the question,

Given: s = 5.10 m, u = 0 m/s ( from rest)

Constant: 9.8 m/s²

Therefore,

v² = 0²+2×9.8×5.1

v² = 99.96

v = √(99.96)

v = 9.99 m/s

As the diver eneters the water,

u = 9.99 m/s, v = 0 m/s

Given: t = 1.34 s

Apply

a = (v-u)/t

a = 9.99/1.34

a = -7.46 m/s²

F = ma.............. Equation 2

Where F = force, m = mass

Given: m = 75 kg, a = -7.46 m/s²,

F = 75(-7.46)

F = -559.5 N

Hence the average force exerted on the diver is 559.5 N

Find the X and Y components of the following:
A. 35 m/s at 57q from the x-axis.

Answers

Explanation:

Given that,

35 m/s at 57° from the x-axis.

Speed, v = 35 m/s

Angle, θ = 57°

Horizontal component,

[tex]v_x=v\cos\theta\\\\=35\times \cos(57)\\\\=19.06 m/s[/tex]

Vertical component,

[tex]v_y=v\sin\theta\\\\v_y=35\times \sin(57)\\\\=29.35\ m/s[/tex]

Hence, this is the required solution.

If you pull with your lower leg such that you exert a 90 N force on the cord attached to your ankle, determine the magnitude of the tension force of your hamstring on your leg and the compression force at the knee joint.

Answers

This question is incomplete, the missing diagram is uploaded along this answer below.

Answer:

- the magnitude of the tension force exerted by the hamstring muscles on the leg is 990 N

- the magnitude of compression force at the knee joint is 900 N

Explanation:

Given the data in the question and diagram below;

Net torque = 0

Torque = force × lever arm

so

F[tex]_{ConF[/tex]  × ( 15.0 in + 1.5 in ) = T[tex]_{HonL[/tex] × 1.5 in

given that F[tex]_{ConF[/tex] = 90 N

90 × ( 15.0 in + 1.5 in ) = T[tex]_{HonL[/tex] × 1.5 in

90 N × 16.5 in =  T[tex]_{HonL[/tex] × 1.5 in

T[tex]_{HonL[/tex] = ( 90 N × 16.5 in ) / 1.5 in

T[tex]_{HonL[/tex] = 990 N

Therefore, the magnitude of the tension force exerted by the hamstring muscles on the leg is 990 N

b) magnitude of compression force at the knee joint;

In equilibrium, net force = 0

along horizontal

F[tex]_{FonB[/tex] - T[tex]_{HonL[/tex] + F[tex]_{ConF[/tex] = 0

we substitute

F[tex]_{FonB[/tex] - 990 + 90 = 0

F[tex]_{FonB[/tex] - 900 = 0

F[tex]_{FonB[/tex] = 900 N

Therefore, the magnitude of compression force at the knee joint is 900 N

A crucible (container) of molten metal has an open top with an area of 5.000 m^2. The molten metal acts as a blackbody radiator. The intensity spectrum of its radiation peaks at a wavelength of 320 nm. What is the temperature of that blackbody?

Answers

Answer:

  T = 9056 K

Explanation:

In the exercise they indicate that the body can be approximated by a black body, for which we can use the Wien displacement relation

                 λ T = 2,898 10⁻³

where lam is the wavelength of the maximum emission

                T = 2,898 10⁻³ /λ

let's calculate

                 T = 2,898 10⁻³ / 320 10⁻⁹

                  T = 9.056 10³ K

                  T = 9056 K

Diwn unscramble the word

Answers

Answer:

WIND Is what you're looking for

Explanation:

The word is WIND

A man pulls his dog (m=20kg) on a sled with a force of 100N at a 60° angle from the horizontal. What is the horizontal component of the force?

A) 100N

B) 196N

C) 50N

D) 86N

show your work please

Answers

Answer:

the horizontal component of the force is 50 N

Explanation:

Given;

force applied by the man, F = 100 N

angle of inclination of the force, θ = 60⁰

mass of the dog, m = 20 kg

The horizontal component of the force is calculated as;

[tex]F_x = F\times cos(\theta)\\\\F_x = 100 \ N \times cos(60^0)\\\\F_x = 100\ N \times 0.5\\\\F_x = 50 \ N[/tex]

Therefore, the horizontal component of the force is 50 N

What is this sport ⚽⚾

Answers

Answer:

sports are all forms of physical activity that contribute to physical fitness, mental well-being and social interaction.

hope it is helpful to you

Một học sinh làm thí nghiệm sóng dừng trên dây cao su dài L với hai đầu A và B cố định . Xét điểm M trên dây sao cho khi sợi dây duỗi thẳng thì M cách B một khoảng a < L/2 . Khi tần số sóng là f = f1 = 60 Hz thì trên dây có sóng dừng và lúc này M là một điểm bụng . Tiếp tục tăng dần tần số thì lần tiếp theo có sóng dừng ứng với f = f2=72 Hz và lúc này M không phải là điểm bụng cũng không phải điểm nút . Thay đổi tần số trong phạm vi từ 73 Hz đến 180 Hz , người ta nhận thấy với f = fo thì trên dây có sóng dừng và lúc này M là điểm nút . Lúc đó , tính từ B ( không tính nút tại B ) thì M có thể là nút thứ ?

Answers

Have suxhebeuxhsbendixbebendue bride. Did e did e end Rudd. R

What is the change in internal energy if 70 J of heat is added to a system and
the system does 30 J of work on the surroundings. Uze al-Q-W.
O A. 40 J
O B. -40.3
O C. 100.
D. -1003

Answers

Answer:

A. 40 J

Explanation:

Given;

heat added to the system, Q = 70 J

work done by the system, W = 30 J

The change in the internal energy of the system is calculated using the first law of thermodynamic as shown below;

ΔU = Q - W

ΔU = 70 J - 30 J

ΔU = 40 J

Therefore, the  change in the internal energy of the system is 40J

A boy pushes his little brother on a sled. The sled accelerates from rest to (4 m/s). If the combined mass of his brother and the sled is (40.0 kg) and (20 W) of power is developéd, how long time does boy push the sled?


16s

300s

15s

23s​

Answers

300 because the mass and weight

The boy pushed the sled for 16 seconds.

We have a boy who pushes his little brother on a sled.

We have to determine for how long time does boy push the sled.

State Work - Energy Theorem.

The Work - Energy theorem states that the work done by the sum of all forces acting on a particle equals the change in the kinetic energy of the particle.

According to the question -

The sled is initially at rest → initial velocity (u) = 0.

Final velocity (v) = 4 m/s

Mass of boy and sled (M) = 40 kg

Power developed (P) = 20 W = 20 Joules/sec

According to work - energy theorem -

Work done (W) = Δ E(K) = E(f) - E(i)

Therefore -

W = ([tex]\frac{1}{2} \times 40 \times 4 \times 4 - \frac{1}{2}[/tex] x 40 x 0) = 320 Joule

Now, Power is defined as the rate of doing work -

P = [tex]\frac{dW}{dt}[/tex] = [tex]\frac{W}{t}[/tex]

20 = [tex]\frac{320}{t}[/tex]

t = 16 seconds

Hence, the boy pushed the sled for 16 seconds.

To solve more questions on Work, Energy and Power, visit the link below -

https://brainly.com/question/208670

#SPJ2

Other Questions
The scatter plot below shows what kind of trend?A positive trendB no trendC random trendD negative trend Which of these terms accurately describes Alzheimer's disease?AutosomalComplex diseaseRecessiveDominant Jason works 22 hours a week and has a pre-tax paycheck of $660 per week. Last week he had to work 34 hours, how much will his pre-tax paycheck be? Ik desperate please help the question is: how many times more people will there be in the town after 15 years than after 10 years??? I need help ASAP!??!?!?!?!? 20 points?!?!?!!pleaseeeee what mass of water will be formed when 3g of hydrogen react with 30g of oxygen? Who wants to help me what is the mean of the data set 8, 8, 9, 11, 24 Tori is cutting fabric squares to make a quilt. Her squares on average are 5 in. on each side with a standard deviation of 0.1 in. If her cuts are normally distributed, what percentage of her squares would be between 4.9 and 5.1 in? 9. Select all expressions that areequivalent to 18a 12.2(9a - 6)6(3a 2)-3(6a + 4)3(6a + 4)0 ( 3(4 6) how can Corruption be eliminated from the society write any six point Do you think white supremacy has been safe through the U.S. government? Which of the following molecules is analcohol? There is only one correct answer.CH3COOHCH3OHCH2NH2C2H5CI What is the area? Bit confused on this one. My First Day on the Job1. In the storys exposition, the narrator reveals that she wants to be a writer but does not know which kind of writer.What does this detail foreshadow?A. The narrator will have a long and difficult job search.B. The narrator will encounter challenges as a journalist.C. The narrator will decide which kind of writer she wants to be.D. The narrator will be offered a job because of her strong writing samples.2. Which excerpt from the story best supports the idea that the narrator admires the people she interviews the first day on the job?A. I wasnt completely sure what I was supposed to be asking besides how the budget cuts would affect each school."B. I realized the answer was simple Reporter 101: Who? What? Where? When? Why? and How?C. It made me think that my new job as a reporter was going to be easy in comparison.D. By the time I finished up, Id gotten a crash course in interviewing and had received an interesting range of opinions about the budget cuts.3. Which excerpt from the story best supports the inference that the narrator is a competent writer?A. . . . Id taken up my boss on his offer and stayed at my part-time job through the summer after college.B. He was happy with the few writing samples Id brought to the interview and offered me the job on the spot.C. . . . I was sitting down with John, one of the executive editors of the newspaper chain, hearing all the daunting details of being a cub reporter.D. Id been an English major and wanted to be a writer, though I had no idea what kind of writer. Cuando se public el poema "mas o menos la muerte" de Mario Benedetti? Complete parts (a) through (c) belowa. A storage pod has a rectangular floor that measures 22 feet by 13 feet and a flat ceiling that is 7 feet above the floor. Find the area of the floor and the volume of the pod.Alap pool has a length of 28 yards, a width of 20 yards, and a depth of 3 yards Find the poof's surface area (the water surface) and the total volume of water that the pool holds Raised flower bed is 30 feet long. 5 feet wide, and 1.2 feet deep. Find the area of the bed and the volume of soil it holdsThe area of the floor of the pod is (Type an integer or a decimal)The volume of the pod is (Type an integer or a decimalb. The pool's surface area is (Type an integer or a decimal)The total volume of the water that the pool holds (Type an integer or a decimal)c. The area of the flower bed is (Type an integer or a decimal)The volume of the soil that the flower bed holds is (Type an integer or a decimal) Does anyone know the answer to this question ? why do you think three ancient societies considered as the cradle of civilization The administrative assistant has 240 cubic inches of paper clips and needs to buy a rectangular prism shaped container to hold them all. He wants a container that is six inches wide and five inches long. Identify the minimum height that would allow the container to hold all of the paper clips. which metals or non-metals are liquid at a room temperature of 25C?